8
$\begingroup$

I need to find all number fields with absolute value of discriminant $\le 20$.

Using Minkovsky's theorem I understood that it should be quadric or cubic extension. The case of quadric is very easy. As for cubic case I understood that it should have 2 complex embeddings and his ideal class group should be trivial. But I don't know to how find all of them.

$\endgroup$
2
  • $\begingroup$ Is there a lower bound on the discriminant? $\endgroup$ May 14, 2013 at 2:02
  • $\begingroup$ Yes, I edited the question. I need the module of discriminant $\le 20$ $\endgroup$
    – Sasha
    May 14, 2013 at 2:12

2 Answers 2

5
$\begingroup$

I'm going to assume you mean that the absolute value of the discriminant has to be $\le 20$. You seem happy with the quadratic case, and have correctly narrowed down the cubic fields to complex cubic fields.

Hint. The trick here is that the smallest value the discriminant of a cubic field can take (in absolute value) is $23$, namely $\mathbb{Q}[x]/(x^3-x^2+1)$ whose discriminant is $-23$, so there are in fact no cubic fields of discriminant $|\Delta|\le 20$.

$\endgroup$
3
  • 1
    $\begingroup$ How can it be proved? It's not obviously for me, that the smallest absolute value of discriminant of cubic field is $23$. $\endgroup$
    – Sasha
    May 14, 2013 at 3:41
  • $\begingroup$ @Sasha How much do you know about the form a cubic field can take, and the relation between the discriminant of the cubic defining the field and the discriminant of the number field itself? That if $K=\mathbb{Q}[x]/(x^3-ax+b)=\mathbb{Q}(\vartheta)$, then $\text{disc}(x^3-ax+b)=4a^3-27b^2=[\mathcal{O}_K:\mathbb{Z}[\vartheta]]^2 \Delta_K$? Minkowski's bound says that $|\Delta_K|\ge 13$, and Stickelberger's theorem says that $\Delta_K\equiv 0,1$ (mod $4$). Can you rule out any of the possibilities? $\endgroup$ May 14, 2013 at 18:13
  • $\begingroup$ I thought about Stickelberger's theorem today. Also it should be complex cubic field, it implies $|\Delta|\le 0%. So there're 4 possibilities $ \Delta = -15,-16,-19,-20.$ I knew this relation between discriminants, but I don't know how it can help me. $\endgroup$
    – Sasha
    May 14, 2013 at 19:20
3
$\begingroup$

As the OP has rightfully seen, it remains to exclude the possibilities $\Delta_K=-15,-16,-19,-20$ for a cubic number field $K$.

We can use class field theory to do this. For example, the cases $-15$ and $-19$ are excluded since they are squarefree and $\mathbb{Q}(\sqrt{-15})$ and $\mathbb{Q}(\sqrt{-19})$ have class numbers not divisible by three (they are $2$ and $1$ respectively). For the details see my answer on this question

Hilbert class field of a quadratic field whose class number is 3

A small modification of the proof in the link also settles the case $-16$: if $\Delta_K=-16$ occurs for a cubic number field $K$ then only $2$ ramifies in $K$ and is in fact wildly ramified, since total ramification would result in $\mathrm{ord}_2(\Delta_K)=2$. But then by the same argument we see that the normal closure $N$ of $K$ yields an unramified Abelian extension of $\mathbb{Q}(\sqrt{-16})=\mathbb{Q}(i)$ of degree $3$, which cannot happen as $\mathbb{Q}(i)$ has class number $1$.

Finally, suppose that $\Delta_K=-20$ for a cubic number field $K$ and let $N$ be its normal closure. As with the previous arguments, since $\mathrm{ord}_5(\Delta_K)=1$ we have non-total ramification of $5$ in $K$, so that $N/\mathbb{Q}(\sqrt{-5})$ is unramified over the prime of $\mathbb{Q}(\sqrt{-5})$ over $5$. Now if $2$ is also non-totally ramified (i.e. wildly ramified) we again see that $N/\mathbb{Q}(\sqrt{-5})$ is an unramified Abelian extension, which cannot happen as $\mathbb{Q}(\sqrt{-5})$ has class number $2$. If $2$ is totally ramified in $K$ then we see that the unique prime $\mathfrak{p}=(2,1+\sqrt{-5})$ in $\mathbb{Q}(\sqrt{-5})$ lying over $2$ ramifies in $N$, and since this ramification is tame (and also the only ramification) we would get that the Abelian extension $N/\mathbb{Q}(\sqrt{-5})$ has conductor $\mathfrak{p}$, so that $N$ would be contained in the ray class field $H_{\mathfrak{p}}$ of $\mathfrak{p}$. However we have the exact sequence $$ 0 \to (\mathcal{O}/\mathfrak{p})^{*}/\mathrm{im}[\mathcal{O}^{*}]\to Cl_{\mathfrak{p}}\to Cl_{\mathbb{Q}(\sqrt{-5})}\to 0, $$ for the ray class group $Cl_{\mathfrak{p}}$, which implies that $[H_{\mathfrak{p}}:\mathbb{Q}(\sqrt{-5})]=2$, which is a contradiction as $[N:\mathbb{Q}(\sqrt{-5})]=3$.

$\endgroup$

You must log in to answer this question.

Not the answer you're looking for? Browse other questions tagged .